Intersection of two vector-valued functions

Click For Summary
To prove that the vector-valued functions r1 and r2 intersect at the point (1,1,3), set r1 equal to r2 and solve for the parameters. The functions are defined as r1(t)=t^2i+tj+3t^3k and r2(s)=(s−1)i+(1/4)s^2j+(5−s)k. It is essential to use different parameters for each function since the same parameter value may not yield the intersection point. The goal is to find the appropriate values of t and s that satisfy both equations, confirming that (1,1,3) lies on both curves. Successfully demonstrating this will establish the intersection of the two vector-valued functions.
karens
Messages
7
Reaction score
0

Homework Statement



Prove that r1 and r2 intersect at (1,1,3).
Let r1 and r2 be defined as:

r1(t)=t^2i+tj+3t^3k
r2(t)=(t−1)i+(1/4)t^2j+(5−t)k

Homework Equations



Intersection is derived from r1=r2.

The Attempt at a Solution



I only formally get to this point and then start messing up. It gets messy and I know I'm not doing the right thing.

r1=r2 --> t^2i+tj+3t^3k = (t−1)i+(1/4)t^2j+(5−t)k
 
Physics news on Phys.org
it may help to write the curves with different parameterisation variables
r1(s)=s^2i+sj+st^3k
r2(t)=(t−1)i+(1/4)t^2j+(5−t)k

then solve for correspdoning intersection point in terms of s & t
 
karens said:

Homework Statement



Prove that r1 and r2 intersect at (1,1,3).
Let r1 and r2 be defined as:

r1(t)=t^2i+tj+3t^3k
r2(t)=(t−1)i+(1/4)t^2j+(5−t)k

Homework Equations



Intersection is derived from r1=r2.

The Attempt at a Solution



I only formally get to this point and then start messing up. It gets messy and I know I'm not doing the right thing.

r1=r2 --> t^2i+tj+3t^3k = (t−1)i+(1/4)t^2j+(5−t)k

Use a different parameter for each vector function. Just because the two curves intersection, there's no guarantee that the same value of the parameter works in both functions.

In this problem, all you need to do is to show that (1, 1, 3) is a point on both curves. In the first function, what value of t gives this point? In the second function what value of s gives the same point?
 
Question: A clock's minute hand has length 4 and its hour hand has length 3. What is the distance between the tips at the moment when it is increasing most rapidly?(Putnam Exam Question) Answer: Making assumption that both the hands moves at constant angular velocities, the answer is ## \sqrt{7} .## But don't you think this assumption is somewhat doubtful and wrong?

Similar threads

  • · Replies 5 ·
Replies
5
Views
1K
  • · Replies 1 ·
Replies
1
Views
1K
Replies
12
Views
6K
  • · Replies 3 ·
Replies
3
Views
2K
Replies
2
Views
3K
  • · Replies 4 ·
Replies
4
Views
2K
  • · Replies 4 ·
Replies
4
Views
2K
  • · Replies 4 ·
Replies
4
Views
23K
Replies
11
Views
4K
  • · Replies 12 ·
Replies
12
Views
3K